Is the answer a? If not can someone explain

Is The Answer A? If Not Can Someone Explain

Answers

Answer 1

Answer:

c . Quadratic binomial

Step-by-step explanation:

a qudratic expression is always in form of ax²+bx + c while a quadratic trinomial is in the form of ax³+bx²+cx+d .

I hope it helps

Answer 2

Answer:

c.) quadratic binomial

Step-by-step explanation:

Quadratic equation:

ax² + bx + c

The polynomial is in this form and like all quadratics forms a parabola, a u-shape, when graphed. Therefore, it is a quadratic equation.

Binomial = an expression with two terms

This polynomial has three terms: x², and 5x, making it a quadratic binomial.


Related Questions

please help as soon as possible
I WILL MARK YOU AS BRAINLIEST ​

Answers

Answer:

Last choice

Step-by-step explanation: Subtract 11 and then divide by 3. A would be greater than -2 and last choice shows that

a) the quadratic x^2-4x-21 can be written in the form (x+a)^2+b find the values of and and b

b) hence, or otherwise, solve the equation x^2-4x-21=0

Answers

Part a. A=-2. B=-25
Part b solutions; x=7 x=-3

evaluate the following expression -7x(7+9)

Answers

Answer:

-112

Step-by-step explanation:

Add

-7(7+9)

Multiply

-7x16

Wallah! You have the answer

-112

[tex]\text{Simplify the expression:}\\\\-7x(7+9)\\\\\text{Use the distributive property}\\\\-49x-63x\\\\\text{Combine like terms}\\\\\boxed{-112x}[/tex]

Pls help me on this................

Answers

Answer:

Step-by-step explanation:

If f(x) = 2x + 1 and g(x) = x-2 what is the value of f(g(f(3)))? This is so confusing plz help me will mark brainliest A) 1 B) 3 C) 5 D) 7 E) 11

Answers

Answer:

11

Step-by-step explanation:

f(3) = 2*3+1 = 6+1 = 7

Then find g(7)

g(7) = 7-2 = 5

Then find f(5)

f(5) = 2*5 +1 = 10+1 = 11

f(g(f(3))) = 11

Answer:

11

Step-by-step explanation:

f(x)=2x + 1

f(3)= 2.3 + 1

f(3)=7

f(g(f(3))) = f(g(7))

Now, we have to find g(7)

g(x)=x -2

g(7)= 7 -2

g(7)=5

f(g(f(3))) = f(g(7)) = f(5)

now we have to find f(5)

f(x)=2x + 1

f(5)=2.5 + 1

f(5)=11

Hope this helps ^-^

Jake is earning money for a new TV. So far, he has put $54.29, $87.20, and $21.44 into his savings account. How much money has Jake saved?

Answers

add all the values together

54.29+87.20=141.49

141.49+21.44=162.93

Answer:

$162.93

Step-by-step explanation:

Do 54.29 + 87.20 + 21.44 and get 162.93 than put this $ in front of you answer  then you have $162.93

Find the solution(s) to x^2- 14x + 49 = 0.
O A. x=-2 and x = 7
B. x= -1 and x = 14
C. x= 7 only
D. x = 7 and x = -7

Answers

Answer:

[tex] \boxed{C. \: x = 7 \: only} [/tex]

Step-by-step explanation:

[tex] = > {x}^{2} - 14x + 49 = 0 \\ \\ = > {x}^{2} - (7 + 7)x + 49 = 0 \\ \\ = > {x}^{2} - 7x - 7x + 49 = 0 \\ \\ = > x(x - 7) - 7(x - 7) = 0 \\ \\ = > (x - 7)(x - 7) = 0 \\ \\ = > {(x - 7)}^{2} = 0 \\ \\ = > x - 7 = 0 \\ \\ = > x = 7[/tex]

Helppppp meeeee answer thissssssss

Answers

Answer:

(-1.025)^3 x (-1.025)^2=(- 1.025)^5

(-y)^3 x (-y)^2 x (-y)= y^6

5= odd, - stays

6= even, - becomes +

Answer:

5) (-1.025)^3* (-1.025)^2 = (-1.025)^5 = (-41/40)^5

10) -y^3 * -y^2 * -y

Multiplying an odd number of negative terms makes the product negative

= - y^3 * y^2 * y

= - y^6

I need help with is question ASAP please

Answers

The y-intercept looks like it's 0 and the slop looks like 6/2 or 3 or maybe 7/2

Is the relation a function?
{(-6, -1), (5,-1), (0, -1), (-2, -1), (3, -1)}

Answers

Answer:

yes it is a function

Step-by-step explanation:

For that no two x values are the same with different y values

Line I and h intersect at what point

Answers

Answer:

please show where the lines are graphed in a picture

Step-by-step explanation:

Which equation represents a line parallel to the line whose equations is -2x + 3y =
-4 and passes through the point (1,3)?

Answers

Answer:

2.  y - 3 = 2/3 (x - 1).

Step-by-step explanation:

-2x + 3y = -4

3y = 2x - 4

y = 2/3 x - 4/3 - so the slope is  2/3.

The slope of a line parallel to it is also 2/3.

It also passes through the point (1, 3).

Using the point-slope form of  a line:

y - y1 = m(x - x1) where m = the slope and (x1, y1) is a point on the line, we have:

y - 3 = 2/3 (x - 1)  <--- is the required equation.

Answer:

2

Step-by-step explanation:

parallel lines have same slope with different intercept

y= mx+b

m is going to be same with different b

so the given function is:

-2x+3y=-43y= 2x-4y= 2/3x - 4/3

Given options:

1. y-3= - 2/3(x-1)  ⇒ y= -2/3x +3 + 2/3 ⇒ y= -2/3x +11/3

it has different slope, so is not parallel

2. y-3= 2/3(x-1) ⇒ y= 2/3x+3-2/3 ⇒ y= 2/3x + 7/3

it has same slope, so is parallelit should be passing through point (1,3)3= 2/3+7/3 ⇒ 3=3, yes it does

3. y-3= -2/3(x+1) ⇒ y= - 2/3x +3- 2/3 ⇒ y= -2/3x + 7/3

it has different slope, is not parallel

4. y-3= 2/3(x+1) ⇒ y= 2/3x +3+ 2/3 ⇒ y= 2/3x +11/3

it has same slope, so is parallelit should be passing through point (1,3)3= 2/3+11/3 ⇒ 3≠13/3, no it doesn't

HELP ASAP it says my questions should be at least 20 characters long so what I’m saying right now doesn’t mean anything.

Answers

Answer: See pic above for answer. I got it from Photomath

Plz put brainliest

Tiffany cells 2 kinds of homemade tomato sauce

Answers

Answer:

b

Step-by-step explanation:

Let x represent the number of quarts of Tuscan sauce and

y represents the number of quarts of marinara sauce Tiffany makes.

A quart of Tuscan sauce requires 6 tomatoes and 1 cup of oil

x quarts requires 6x tomatoes and 1x cups of oil

A quart of her marinara sauce requires 5 tomatoes and 1.25 cups of oil

y quarts requires 5y tomatoes and 1.25 y cups of oil

She has 45 tomatoes and 10 cups of oil on hand.

So the constraints are

6x+5y≤45

1x+1.25y≤10

x>=0 and y>=0

After drawing a diagram of what's required to make 1 quart of Tuscan and 1 quart of Marinara sauce, you can more clearly choose answer choice (B) 6x + 5y < (or equal to) 45, x + 5/4y < (or equal to) 10, x>(or equal to)0, y>(or equal to)0.

please mark brainliest :)

the answer is B !!! hope it’s correct

What is -6 divided by -1.2?

Answers

Answer:

5.

Step-by-step explanation:

This is the same as saying:

-6/(-6/5).

To solve the equation, we multiply by the reciprocal and simplify. In this case, we multiply both sides of the equation by -5/6.

-6 * (-5/6) = 5.

The answer must be positive, as two negatives divided equal a positive.

I hope this helps!

The solution of expression after divide is,

⇒ 5

We have to given that;

⇒ - 6 is divided by - 1.2

Now, We can simplify as,

⇒ - 6 is divided by - 1.2

It can be written as,

⇒ - 6 ÷ - 1.2

⇒ - 6 / - 1.2

⇒ 6 / 1.2

⇒ 60 / 12

⇒ 5

Thus, The solution of expression after divide is,

⇒ 5

Learn more about the divide visit:

https://brainly.com/question/28119824

#SPJ6

A few years ago, a country consumed 19,400,000 barrels of oil per day. That same year, there were about 119,000,000 households in that country. A barrel of oil is 42 U.S. gallons. Estimate how many gallons each household in the country consumed each day that year by rounding down all three numbers. (Round 19,400,000 barrels to 19,000,000 barrels, 119,000,000 households to 115,000,000 households, and 42 gallons to 40 gallons. Round your answer to two decimal places.)

_________ gal/household/day

Calculate the answer without using estimation and compare the two results. (Round your answer to two decimal places.)
___________gal/household/day

Answers

Following are the calculation to fill in the blank:

For Part a)

Each home in this town consumed

[tex]\to 40 \times \frac{19000000}{115000000}\\\\\to 40 \times \frac{19}{115}\\\\\to 40 \times 0.165\\\\\to 6.6 \ \text{gallons of oil per day}[/tex]

For Part b)

Without completing the sentence,

[tex]\text{Total consumption = 19200000 barrels}\\\\\text{Total number of households = 118000000}[/tex]

As a result, each household consumed.

[tex]\to \frac{19200000}{118000000} \\\\\to \frac{192}{1180} \ (barrels)\\\\[/tex]

In the given scenario, each barrel includes 42 gallons of oil. Therefore, every household consumed [tex]42 \times \frac{192}{1180} = 6.83 \ gallons[/tex] oil each day.

Learn more:

brainly.com/question/14798687

Hence, each household will consume 6.60 gallons per day.

Total Households of that country =115000000 (approx.)

Total barrels used per day =19000000 (approx.)

Given 1 barrel = 40 gallons (approx.)

So total gallons used per day = 19000000 x 40 =760000000

So each household will consume gallons per day = total gallons used per day / Total household of that country

=[tex]=\frac{760000000}{ 11500000}[/tex]

=6.60 gallons/household/day

So each household will consume 6.60 gallons per day.

Learn More: https://brainly.com/question/12272570

What is the midpoint of the segment below? (2,3)(-3,-2)

Answers

Answer:

(-0.5, 0.5)

Step-by-step explanation:

If those two points are endpoints, just use the midpoint formula.

(-0.5, 0.5)

Answer:

(-1/2,1/2)

Step-by-step explanation:

To find the midpoint, add the x coordinates and divide by 2

(2+-3)/2 = -1/2

add the y coordinates and divide by 2

(3+-2)/2 = 1/2

The midpoint is (-1/2,1/2)

00:00
Brice is finding the sum of 468 and 241 by breaking it into smaller problems
He uses place value and finds the sums of the hundreds, tens, and ones.
What is the sum of the tens? Enter your answer in the box.
1​

Answers

468+241=709 so 0 is in the tenth place because 6+4= 10 and one gets carried over to the 4+2

14. Roger is on a playground swing, and he is swinging back and forth in such a way that the height, h, in feet, of the swing off the ground is given by the equation h=3cos(3π/2t) +5, where t is in seconds. How many seconds elapses between two consecutive times that the swing is at its maximum height?​

Answers

Answer:

The time [tex]t = \frac{3}{2}[/tex]  seconds elapses between two consecutive times that the swing is at its maximum height  'h' = 2

 Step-by-step explanation:

Explanation:-

Step(i):-

Given function [tex]h(t) = 3 cos (\frac{3\pi }{2 t} ) +5[/tex] ....(i)

By using derivative formulas

[tex]\frac{d cosx }{d x} = -sinx[/tex]

[tex]\frac{d x^{n} }{d x} = n x^{n-1}[/tex]

[tex]\frac{d t^{-1} }{d x} = -1 t^{-1-1} = - t^{-2} = \frac{-1}{t^{2} }[/tex]

Step(ii):-

Differentiating equation(i) with respective to 't'

[tex]h^{l} (t) = 3(-sin(\frac{3\pi }{2t})\frac{d}{dt} (\frac{3\pi }{2t } )+0[/tex]  ...(ii)

[tex]h^{l} (t) = 3(-sin(\frac{3\pi }{2t})(\frac{-3\pi }{2t^{2} } )+0[/tex]

Equating zero

[tex]h^{l} (t) = 3(-sin(\frac{3\pi }{2t})(\frac{-3\pi }{2t^{2} } )=0[/tex]

[tex]3(-sin(\frac{3\pi }{2t})(\frac{-3\pi }{2t^{2} } ) = 0[/tex]

on simplification , we get

[tex](sin(\frac{3\pi }{2t}) = 0[/tex]

now we use formulas

sin 0 = 0 and sinπ = 0

General solution

[tex](sin(\frac{3\pi }{2t}) = sin\pi[/tex]

[tex](\frac{3\pi }{2t}) = \pi[/tex]

Cancellation 'π' on both sides, we get

[tex]3 = 2 t[/tex]

Dividing '2' on both sides , we get

[tex]t = \frac{3}{2}[/tex]

Again differentiating with respective to 't' , we get  

[tex]h^{ll} (t) = 3(-cos(\frac{3\pi }{2t})(\frac{-3\pi }{2t^{2} } )+ (-3)(-sin(\frac{3\pi }{2t} )(\frac{6\pi }{2t^{3} }[/tex]

Put t= 3/2 and simplification

[tex]h^{ll} (t) < 0[/tex]

The maximum height

                   [tex]h(t) = 3 cos (\frac{3\pi }{2 t} ) +5[/tex]

               [tex]h(\frac{3}{2} ) = 3 cos (\frac{3\pi }{2(\frac{3}{2} )} )+5[/tex]

              [tex]h(\frac{3}{2} ) = 3 cos (\pi )+5 = -3+5 =2[/tex]

[tex]t = \frac{3}{2}[/tex]  seconds elapses between two consecutive times that the swing is at its maximum height  'h' = 2

Conclusion:-

The time [tex]t = \frac{3}{2}[/tex]  seconds elapses between two consecutive times that the swing is at its maximum height  'h' = 2

 

 

If g(x) = -4x + 5, find g(2x-1).

Answers

Answer:

- 8x + 9

Step-by-step explanation:

To evaluate g(2x - 1) substitute x = 2x - 1 into g(x), that is

g(2x - 1)

= - 4(2x - 1) + 5 ← distribute parenthesis and simplify

= - 8x + 4 + 5

= - 8x + 9

Answer:

= - 8x + 9

Step-by-step explanation:

= - 8x + 4 + 5

= - 8x + 9

What is the equation in slope-intercept form of a line with slope of 5 and y-intercept of 3?

Answers

y=5x+3

Because the starting value(y intercept is 3) and the slope(x) is 5

Please Help me it’s geometry

Answers

Answer:

Option C.

Step-by-step explanation:

Given: In [tex]\Delta OPQ,m\angle O=107^{\circ},m\angle P=28^{\circ} [/tex].

In [tex]\Delta OPQ[/tex],

[tex]m\angle O+m\angle P+m\angle Q=180^{\circ}[/tex]    (Angle sum property)

[tex]107^{\circ}+28^{\circ}+m\angle Q=180^{\circ}[/tex]

[tex]135^{\circ}+m\angle Q=180^{\circ}[/tex]

[tex]m\angle Q=180^{\circ}-135^{\circ}[/tex]

[tex]m\angle Q=45^{\circ}[/tex]

Now,

[tex]m\angle O>m\angle Q>m\angle P[/tex]

In a triangle, the greatest angle has largest opposite side and smallest angle has smallest opposite side. So, we conclude that

[tex]PQ>OP>QO[/tex]

Therefore, the correct option is C.

Becky made 4 quarts of chicken noodle soup in a big pot, then served 112-cup bowls of soup to 6 people. How much soup is left in the pot?

Answers

Answer:

1qt 3c

Step-by-step explanation:

Micheal has 1/2 of a yard of fabric to make book covers. Each book cover is made from 1/8 of a yard of fabric. How many book covers can Micheal make?

Answers

Answer:

4

Step-by-step explanation:

[tex]\frac{1}{2}[/tex] ÷ [tex]\frac{1}{8}[/tex] = 4



A new car is purchased for 23400 dollars. The value of the car depreciates at 11.5% per year. To the nearest year, how long will it be until the value of the car is 12700 dollars?

Answers

Answer: 5 years

Step-by-step explanation:

Using same formula as for Compound Interest:

12700 = 23400 (1-.115)^t

12700 = 23400 x 0.885^t

0.885^t = 127/234

Convert decimal into fraction:

177/200^t = 127/234

Take logarithm of both sides of thr equation:

t = log 177/200 (127/234)

t = 5.00242

Find the value of x in the triangle pair below

Answers

Answer:

x=16.97056274... or 12[tex]\sqrt{2}[/tex]

Step-by-step explanation:

the hypotenuse of a 45, 45, 90 triangle is the length of the leg times the square root of 2


[tex]( {2}^{ - 1} + {3}^{ - 1} )^{2} [/tex]
solve.

will give the brainliest​

Answers

Answer:

25/36

Step-by-step explanation:

(2^-1 + 3^-1)^2

(1/2 + 1/3)^2

(5/6)^2 = 25/36

25/36 or .694

2^-1= 1/2
3^-1=1/3

Add the together
5/6 square it and you get 25/36

BEST ANSWER GETS TO CHOOSE BRAINLESST OR FOLLOW!​

Answers

Answer:13 is>

14 is =

15 is >

16 is<

17 is<

18 is =

Step-by-step explanation:

I just know plz mark brainliest

PLEASE HELP WILL GIVE BRAINLIEST AND 20 POINTS

solve this system of equations

3y-5x=12
y=1/3x

Answers

Answer:

x=-3

y=-1

(-3,-1)

Answer:

(-3, -1).

Step-by-step explanation:

I have attached the work to your problem.

Please see the attachment below.

I hope this helps!

A. 256
B. 265
C. 297
D. 279​

Answers

Answer:

A.

Step-by-step explanation:

So calculate one of the triangles.

[tex]\frac{12*8}{2} =48[/tex]

48 * 4 = 192

192 + 64 = 256

Other Questions
Why did the author begin the article by describing some of the causes of WWII? A nursery located in Florida uses a water meter to measure its water usage 1 pointin CCF. It is charged $0.36 per CCF of water. Its last meter reading was123,692 and their present reading is 127,394. For this usage, the nursery wascharged $332.72*True or false? The First National Bank of Nelsonville has no excess reserves when a new deposit of $10,000 is made. The required reserve ratio for all banks is 5 percent. How much is the largest possible increase in checking account balances throughout the entire banking system make sentences of these words first as transitive and then as intransitive:drink soilformsingplayplanmove writereadhear plzzz answer me. 3 ways in which a country address or improve the problem of illegal immigration. Simon is supposed to find a fictional text to compare with the nonfiction book about Italian artist that he read which text would best match the purpose What is the factorization of the polynomial below?- x2 - 15x - 56A. (X + 8)(x + 7)B. (x+3)(x - 7)C. -1(x+8)(x + 7)D. (-x+3)(x-7) Give two examples of spatial interference which can be easily observed. Small explanation of each object please, just so I can understand. A study will be conducted to examine a new medicine intended to reduce high blood pressure in adult men who have high blood pressure. As part of the study, a random sample of 40 men with high blood pressure will have their blood pressure measured, and then they will take the new medicine every day for one month. At the end of the month, their blood pressure will be measured again. Of the following, which is the best procedure to investigate whether there will be convincing statistical evidence of a change, on average, in blood pressure for men with high blood pressure who take the new medicine? (A) A one-sample z-test for a proportion (B) A two-sample z-test for a difference between proportions (C) A two-sample i-test for the difference between two means (D) A matched-pairs t-test for a mean difference (E) A chi-square test of independence According to supply-side fiscal policy, increasing tax rates on wages and profits will: Group of answer choices result in higher unemployment and inflation. reduce both unemployment and the price level. lower the price level but raise unemployment. increase the real output as well as the price level. Loyalists lived throughout the colonies, but their numbers were smallest, around (1%, 5%, 10%, 15%) , in places like (New Jersey, New York, Massachusetts, Georgia), where patriots had long pursued campaigns of political education and mobilization. Areas where (young people, native-born colonists,new British immigrants, Germans) clustered, however, tended to be loyalist strongholds. They were concentrated in (Rhode Island, Vermont, Virginia, Georgia), New York, and the Carolina Piedmont. Elites in (Quebec, New Brunswick, Louisiana, Nova Scotia) also supported the Crown and formed a significant minority. The line AB has midpoint (2, 5).A has coordinates (1, 7).Find the coordinates of B. wright a essay why you think snape in harry potter is a death eater. Find the volume of the prism. Round to the nearest tenth if necessary How many molecules are in 3.8 mols of copper? The World Health Organization defines health as the absence of disease or infirmity. True False please this is just asking dont get mad but what do you think about the protest Cooling down after a workout allows the oxygen to continuing moving through the body and keeps the muscles from tightening up to fast. True False help plzzzzzzzzzzzz what is the equation of the line of best fit for the following data (4,3) (6,4) (8,9) (11,12) (13,17)